General Awareness - SPLessons

UPSC Current Affairs Prelims Quiz

Home > > Tutorial
SPLessons 5 Steps, 3 Clicks
5 Steps - 3 Clicks

UPSC Current Affairs Prelims Quiz

shape Introduction

In General Current Affair means General Issue. These Current Affairs related to several fields like International, National, Sports, Politics, Economy, Science and Technology, and Environment. Current Affairs Section is very important in several exams like UPSC, SSC, Banking, RRB, SBI, IAS, and Insurance etc. Through daily newspapers and daily news, there is a huge chance of scoring maximum marks in several exams.
The main aim of the article UPSC Current Affairs Prelims Quiz is to clear the civil exams. The Civil's Exam is the toughest exam in competitive Field. It provides important and useful questions related to Current Affairs.

shape Quiz

1. Consider the following statements about Sangai Deer: 1. It is the state animal of Meghalaya. 2. It is categorized as endangered as per IUCN list. Which of the above statements is/are correct:
    A. 1 Only B. 2 Only C. Both 1 and 2 D. None

Answer - Option B
Explanation:
The sangai is an endemic and endangered subspecies of brow- antlered deer found only in Manipur, India. It is found in its natural habitat only at Keibul Lamjao National Park over the floating biomass locally called “phumdi” in the south eastern part of Loktak Lake. Its IUCN status: Endangered. Every year the State of Manipur celebrates the “Manipur Sangai Festival” from 21st to 30th November.
The ‘Festival’ is named after the State animal, Sangai, the brow- antlered deer found only in Manipur. It started in the year 2010 and has grown over the years into a big platform for Manipur to showcase its rich tradition and culture to the world.
The festival will reflect the State’s proud cultural heritage and the love for art which is inherent amongst various tribes inhabiting the State of Manipur. The State’s classical dance form, ‘Ras Leela’ is quite famous all over the world for its distinctiveness from any other dance forms in India.
2. Consider the following statements about Char Dhams: 1. These are 4 pilgrimage centers of Hinduism. 2. The names of these centres are: Yamunotri, Gangotri, Dehradun, Kedarnath. Which of the above statements is/are correct:
    A. 1 Only B. 2 Only C. Both 1 and 2 D. None

Answer - Option A
Explanation:
About Chardham project:
The project involves developing and widening 900-km of national highways connecting the holy Hindu pilgrimage sites of; Badrinath, Kedarnath, Gangotri, and Yamunotri at an estimated cost of Rs.12,000 crores. The highway will be called Char Dham Mahamarg (Char Dham Highway) and the highway construction project will be called as Char Dham Mahamarg Vikas Pariyojana (Char Dham Highway Development Project).
The roads will be widened from 12m to 24m and the project will involve construction of tunnels, bypasses, bridges, subways and viaducts.
3. Which of the following airports was recently renamed as “Veer Sai Airport”:
    A. Chatrapati Shivaji Airport, Maharashtra B. Madras Airport, Tamil Nadu C. Varanasi Airport, U.P D. Jharsugoda Airport, Odisha

Answer - Option D
Explanation:
The Union Cabinet has approved renaming of Jharsuguda Airport in Odisha as “Veer Surendra Sai Airport, Jharsuguda”.
Who was Veer Surendra Sai?
Veer Surendra Sai who is well-known freedom fighter and a tribal leader of Odisha.
Born in 1809 in Khinda in Sambalpur, he was direct descendant of Madhukar Sai and was legally entitled to be crowned as king of Sambalpur after demise of king Maharaja Sai in 1827. But he was not acceptable to British power and ignored his claim for succession. He revolted against the British for throne after it allowed widow of Madhukar Sai Rani Mohan Kumari to succeed him and then followed by succession of Narayan Singh, a descendant of royal family but born of low caste as king of Sambalpur.
The aim of Surendra Sai’s revolt was to drive the British out of Sambalpur. His revolution against the British commenced from 1827 when he was only 18 years of age and continued till 1862 when he surrendered and even after that, until he was finally arrested in 1864.
4. Sentinelese Tribe was recently in news. The tribe is found in:
    A. Jharkhand B. Andaman Islands C. Chhattisgarh D. Madhya Pradesh

Answer - Option B
Explanation:
The Sentinelese are a negrito tribe who live on the North Sentinel Island of the Andamans. The inhabitants are connected to the Jarawa on the basis of physical, as well as linguistic similarities. Their numbers are believed to be less than 150 and as low as 40.
5.Consider the following statements about SHe-box: 1. It is an online portal to file complaints relating to Sexual Harassment at Workplace. 2. Portal is developed and managed by Ministry of Women and Child Development Which of the above statements is/are correct:
    A. 1 Only B. 2 Only C. Both 1 and 2 D. None

Answer - Option C
Explanation:
What is it? It is an online complaint management system for registering complaints related to sexual harassment at workplace. It was launched by the Ministry of Women and Child Development. The complaint management system has been developed to ensure the effective implementation of Sexual Harassment of Women at Workplace (Prevention, Prohibition and Redressal) Act (the SH Act), 2013.
How it works? Once a complaint is submitted to the portal, it will be directly sent to the Internal Complaints Committee (ICC) of the concerned Ministry/Department/PSU/Autonomous Body etc. having jurisdiction to inquire into the complaint. Through this portal, WCD as well as complainant can monitor the progress of inquiry conducted by the ICC.
6. Consider the following statements: 1. Every Year, 25 November is celebrated as International Day for the Elimination of Violence against Women. 2. The theme of 2018 was: “Pink the World: #HearMeToo. Which of the above statements is/are correct:
    A. 1 Only B. 2 Only C. Both 1 and 2 D. None

Answer - Option A
Explanation:
International Day for the Elimination of Violence against Women (IDEVAW) is observed every year across the world on 25 November.
Theme: “Orange the World: #HearMeToo”.
Background:
International Day for the Elimination of Violence against Women was instituted by United Nations General Assembly (UNGA) in December 1999.
This day is commemorated in memory of Mirabal sisters who were three political activists from Dominican Republic. They were brutally assassinated during the Rafael Trujillo dictatorship (1930-1961) in 1960.
7. Consider the following statements about Nirbhaya Fund: 1. It is managed by Ministry of Women and Child Development. 2. It is instituted to ensure safety and security of Women. Which of the above statements is/are correct:
    A. 1 Only B. 2 Only C. Both 1 and 2 D. None

Answer - Option B
Explanation:
In order to eradicate violence against women and girls, Central Government has set up Nirbhaya Fund.
It is administered by the Department of Economic Affairs, Ministry of Finance.
It can be utilized for projects for women safety and security.
Ministry of Women and Child Development is the nodal authority for appraising/recommending the proposals/schemes to be funded under Nirbhaya fund.
8. Consider the following statements with reference to RIMES: 1. It is a UN registered international and inter-governmental institution. 2. Owned and managed by its 48 member nations, it aims to build capacities in multi-hazard early warning systems. Which of the above statements is/are correct:
    A. 1 Only B. 2 Only C. Both 1 and 2 D. None

Answer - Option C
Explanation:
The Odisha government has signed a Memorandum of Understanding (MoU) with the Regional Integrated Multi-Hazard Early Warning System (RIMES) for effective management of disasters.
RIMES was established on 30 April 2009 to provide user-relevant early warning services to its Member States and others.
It is a UN registered international and inter-governmental institution.
It is owned and managed by its 48 members and collaborating states for building capacities in the generation and application of user- relevant early warning information
The programme unit of the agency is located in Thailand. At present, India is chairing RIMES.
9. Consider the following statements: 1. The Constitution of India was adopted on 26 November, 1949. 2. 26 November is celebrated as “Samvidhaan Diwas” to mark this day. Which of the above statements is/are correct:
    A. 1 Only B. 2 Only C. Both 1 and 2 D. None

Answer - Option C
Explanation:
Constitution day which is also known as the Samvidhan Divas is celebrated every year on November 26 to mark the day on which the Constitution of India was adopted. While the adoption of the Constitution took place on November 26, 1949, it came into effect on January 26, 1950.
The draft of the constitution was prepared by the drafting committee under BR Ambedkar’s aegis. According to the government notification, the Constitution Day was also a tribute to Ambedkar.
Earlier, this day was commemorated as National Law Day, after a resolution by the Supreme Court Bar Association, a lawyers’ body, in 1979.
10. Consider the following statements about Competition Commission of India:
    A. 1 Only B. 2 Only C. Both 1 and 2 D. None

Answer - Option C
Explanation:
About Competition Commission of India:
The Competition Commission of India (CCI) was established under the Competition Act, 2002 for the administration, implementation and enforcement of the Act, and was duly constituted in March 2009. Chairman and members are appointed by the central government.
The following are the objectives of the Commission:
  • To prevent practices having adverse effect on competition.
  • To promote and sustain competition in markets.
  • To protect the interests of consumers.
  • To ensure freedom of trade.
1. National Tiger Conservation Authority is a/an:
    A. It is an executive body attached MoEF & CC. B. Statutory Body C. Constitutional Body D. A society registered under Societies Registration Act 1860

Answer - Option B
Explanation:
The National Tiger Conservation Authority is a statutory body under the Ministry of Environment, Forests and Climate Change constituted under enabling provisions of the Wildlife (Protection) Act, 1972, as amended in 2006, for strengthening tiger conservation, as per powers and functions assigned to it under the said Act.
The functions of NTCA are as follows:
Ensuring normative standards in tiger reserve management
Preparation of reserve specific tiger conservation plan
Laying down annual/ audit report before Parliament
Instituting State level Steering Committees under the Chairmanship of Chief Minister and establishment of Tiger Conservation Foundation.
According approval for declaring new Tiger Reserves.
2. Consider the following statements about Dredging Corporation of India: 1. Recently, Government of India approved 100% Sale of Government Shares in it. 2. Prior to this, it was a Miniratna PSU. Which of the above statements is/are correct:
    A. 1 Only B. 2 Only C. Both 1 and 2 D. None

Answer - Option C
Explanation:
Cabinet Committee on Economic Affairs (CCEA) has approved strategic disinvestment of 100% Government of India’s shares in Dredging Corporation of India Limited (DCIL) to consortium of four ports.
The consortium of four ports consists of Vishakhapatnam Port Trust (Andhra Pradesh), Paradeep Port Trust (Odisha), Jawahar Lal Nehru Port Trust (Maharashtra) and Kandla Port Trust (Gujarat).
It is miniratna public sector unit (PSU) engaged in the business of dredging.
It was established in March 1976 and is headquartered in Visakhapatnam, Andhra Pradesh.
It reports to the Ministry of Shipping.
It does dredging for Indian seaports exclusively. It is involved in capital dredging, beach nourishment, and land reclamation.
3. . Consider the following statements about National Green Tribunal: 1. It is a statuary body established under the National Green Tribunal Act 2010. 2. Currently, the tribunal has 10 Judicial and 10 Expert Members. Which of the above statements is/are correct:
    A. 1 Only B. 2 Only C. Both 1 and 2 D. None

Answer - Option C
Explanation:
Justice Adarsh Kumar Goel has been appointed the chairperson of the National Green Tribunal (NGT).
Chairman: is the administrative head of the tribunal, also serves as a judicial member and is required to be a serving or retired Chief Justice of a High Court or a judge of the Supreme Court of India.
Selection: Members are chosen by a selection committee (headed by a sitting judge of the Supreme Court of India) that reviews their applications and conducts interviews. The Judicial members are chosen from applicants who are serving or retired judges of High Courts.
Expert members are chosen from applicants who are either serving or retired bureaucrats not below the rank of an Additional Secretary to the Government of India (not below the rank of Principal Secretary if serving under a state government) with a minimum administrative experience of five years in dealing with environmental matters.
Or, the expert members must have a doctorate in a related field.
The Tribunal is not bound by the procedure laid down under the Code of Civil Procedure, 1908, but shall be guided by principles of natural justice.
The Tribunal is mandated to make and endeavour for disposal of applications or appeals finally within 6 months of filing of the same.
4. Consider the following statements about Regional Connectivity Scheme: 1. The purpose of the scheme is to connect unserved regions by air transport and to make air transport affordable to Common Citizens. 2. Ministry of Tourism is the nodal ministry for the scheme. Which of the above statements is/are correct:
    A. 1 Only B. 2 Only C. Both 1 and 2 D. None

Answer - Option A
Explanation:
UDAN (Ude Desh Ka Aam Nagrik) is the Government’s initiative to make air travel to India’s tier II and tier III cities affordable to the aam aadmi.
The idea is to put smaller cities and remote regions on the aviation map, by getting domestic airlines to ply more regional routes.
Under the scheme, the Government offers incentives to airlines to flag off new flights to neglected smaller cities and towns by providing Viability Gap Funding to make these operations profitable.
Airlines are required to bid for exclusive rights to fly on the regional routes opened up under the scheme. They must sell a specific number of seats on each flight at a fixed fare of 2,500 for one hour of flying. In the case of helicopter operations, allowed for the first time now, fares are capped at 2,500 for a 30-minute flight. Objectives of the scheme:
The primary objective of RCS is to facilitate / stimulate regional air connectivity by making it cheap and affordable.
Promoting affordability of regional air connectivity is envisioned under RCS by supporting airline operators through: Concessions and Financial (viability gap funding or VGF) support.
5. 15. Consider the following statements about Namami Gange Programme: 1. The programme’s purpose is to clean River Ganges in an effective and coordinated manner by involving all stakeholders. 2. For the purpose of Project implementation, a three tier structure has been created at Central, State and District level. Which of the above statements is/are correct:
    A. 1 Only B. 2 Only C. Both 1 and 2 D. None

Answer - Option C
Explanation:
Namami Gange programme was launched as a mission to achieve the target of cleaning river Ganga in an effective manner with the unceasing involvement of all stakeholders, especially five major Ganga basin States – Uttarakhand, Uttar Pradesh, Jharkhand, Bihar and West Bengal.
The programme envisages: River Surface Cleaning, Sewerage Treatment Infrastructure, River Front Development, Bio-Diversity, Afforestation and Public Awareness
The program would be implemented by the National Mission for Clean Ganga (NMCG), and its state counterpart organizations i.e., State Program Management Groups (SPMGs). In order to improve implementation, a three-tier mechanism has been proposed for project monitoring comprising of a) High level task force chaired by Cabinet Secretary assisted by NMCG at national level, b) State level committee chaired by Chief Secretary assisted by SPMG at state level and c) District level committee chaired by the District Magistrate.
The program emphasizes on improved coordination mechanisms between various Ministries/Agencies of Central and State governments.
6. India has Strategic Petroleum Reserves Programme to store crude oil in dedicated caverns to provide cushion against any external supply disruptions. Which of the following places will have such caverns: 1. Vishakhapatnam 2. Mangalore 3. Padur 4. Chandikhole 5. Bikaner
    A. 1, 3 and 5 Only B. 1, 2 and 3 Only C. 2, 3 and 4 Only D. All are Correct

Answer - Option D
Explanation:
To ensure energy security, the Government of India had decided to set up 5 million metric tons (MMT) of strategic crude oil storages at three locations namely, Visakhapatnam, Mangalore and Padur (near Udupi).
These strategic storages would be in addition to the existing storages of crude oil and petroleum products with the oil companies and would serve as a cushion during any external supply disruptions.
In the 2017-18 budget, it was announced that two more such caverns will be set up Chandikhole in Jajpur district of Odisha and Bikaner in Rajasthan as part of the second phase.
The construction of the Strategic Crude Oil Storage facilities is being managed by Indian Strategic Petroleum Reserves Limited (ISPRL), a Special Purpose Vehicle, which is a wholly owned subsidiary of Oil Industry Development Board (OIDB) under the Ministry of Petroleum & Natural Gas
7. Energy Efficiency Services Limited was recently seen in news. It is:
    A. An association of world organisations working towards energy conservation. B. A U.N body C. A Government of India body under the control of Ministry of Power. D. An NGO working towards sustainable practices in energy use.

Answer - Option C
Explanation:
Energy Efficiency Services Limited (EESL), under the administration of Ministry of Power, Government of India, is working towards mainstreaming energy efficiency and is implementing the world’s largest energy efficiency portfolio in the country.
Driven by the mission of Enabling More – more transparency, more transformation, and more innovation, EESL aims to create market access for efficient and future-ready trans-formative solutions that create a win-win situation for every stakeholder. By 2020, EESL seeks to be a US$ 1.5 billion (INR 10,000 crore) company.
8. Consider the following statements about Yuva Sahakar Scheme: 1. The scheme has been launched by Ministry of Youth Affairs. 2. The purpose of the scheme is to attract youth to cooperative ventures. 3. The scheme is linked to Rs1000 Crore Cooperative Start-up and Innovation Fund. Which of the above statements is/are correct:
    A. 1 and 2 Only B. 2 and 3 Only C. 1 and 3 Only D. All are Correct

Answer - Option B
Explanation:
To cater to the needs and aspirations of the youth, the National Cooperative Development Corporation (NCDC) has come up with a youth-friendly scheme ‘Yuva Sahakar-Cooperative Enterprise Support and Innovation Scheme” for attracting them to cooperative business ventures.
Key facts:
The scheme would encourage cooperatives to venture into new and innovative areas.
NCDC has created a dedicated fund with liberal features enabling youth to avail the scheme. The scheme will be linked to Rs 1000 crore ‘Cooperative Start-up and Innovation Fund (CSIF)’ created by the NCDC. It would have more incentives for cooperatives of North Eastern region, Aspirational Districts and cooperatives with women or SC or ST or PwD members.
The funding for the project will be up to 80% of the project cost for these special categories as against 70% for others. The scheme envisages 2% less than the applicable rate of interest on term loan for the project cost up to Rs 3 crore including 2 years moratorium on payment of principal. All types of cooperatives in operation for at least one year are eligible.
9. Consider the following statements about National Cooperative Development Corporation: 1. It is an executive body set up in 1963. 2. The corporation is under the control of Ministry of Agriculture and Farmers’ Welfare. Which of the above statements is/are correct:
    A. 1 Only B. 2 Only C. Both 1 and 2 D. None

Answer - Option B
Explanation:
National Cooperative Development Corporation (NCDC) was established by an Act of Parliament in 1963 as a statutory Corporation under Ministry of Agriculture & Farmers Welfare.
It has many regional centres to provide the financial assistance to Cooperatives/Societies/Federations.
10. With reference to Swadesh Darshan Scheme, consider the following statements:
    A. 1 Only B. 2 Only C. Both 1 and 2 D. None

Answer - Option C
Explanation:
The Tourism Ministry had launched ‘Swadesh Darshan’ scheme with an objective to develop theme-based tourist circuits in the country.
The scheme is 100% centrally funded for the project components undertaken for public funding.
To leverage the voluntary funding available for Corporate Social Responsibility (CSR) initiatives of Central Public Sector Undertakings and corporate sector.
Funding of individual project will vary from state to state and will be finalised on the basis of detailed project reports prepared by PMC (Programme Management Consultant).
A National Steering Committee (NSC) will be constituted with Minister in charge of M/O Tourism as Chairman, to steer the mission objectives and vision of the scheme.
A Mission Directorate headed by the Member Secretary, NSC as a nodal officer will help in identification of projects in consultation with the States/ UTs governments and other stake holders.
PMC will be a national level consultant to be appointed by the Mission Directorate.
1. ACROSS Scheme is an initiative of:
    A. Ministry of External Affairs B. Ministry of Corporate Affairs C. Ministry of Earth Sciences D. Ministry of Parliamentary Affairs

Answer - Option C
Explanation:
The Cabinet Committee has approved continuation of the nine sub-schemes of the umbrella scheme “Atmosphere & Climate Research Modelling Observing Systems & Services (ACROSS)” during 2017- 2020.
Background:
The Ministry of Earth Sciences has a mandate to carry out research and development activities to develop and improve capability to forecast weather, climate and natural hazard related phenomena. Towards this direction, MoES has taken several initiatives to formulate specific schemes like weather and climate modelling, monsoon research, climate change science & climate services etc.
These schemes involve multi-institutes wherein each unit has a designated role for accomplishing the aforesaid tasks. As a result, all these schemes with specific objectives and budget are implemented in an integrated manner and are put together under the umbrella scheme “ACROSS”.
ACROSS Scheme:
ACROSS scheme pertains to the atmospheric science programs of the Ministry of Earth Sciences (MoES).
It addresses different aspects of weather and climate services, which includes warnings for cyclone, storm surges, heat waves, thunderstorms etc.
Each of these aspects is incorporated as nine sub-schemes under the umbrella scheme “ACROSS” and is implemented in an integrated.
The ACROSS scheme consists of nine sub-programmes which are multidisciplinary and multi institutional in nature and will be implemented in an integrated manner.
2. Consider the following observations about India by a report recently released by International Vaccine Access Center: 1. India faced highest number of deaths of Children under 5 Years of age due to Pneumonia and Diarrhoea in 2016. 2. India has achieved universal vaccination for Pneumonia and Diarrhoea. Which of the above statements is/are correct:
    A. 1 Only B. 2 Only C. Both 1 and 2 D. None

Answer - Option A
Explanation:
Had there been universal vaccination, we would not have the distinction of the country having highest number of deaths of Children due to Pneumonia and Diarrhoea. Highlights of the report:
Even as vaccine coverage has significantly improved in India, other interventions against pneumonia and diarrhoea, the two major causes of child deaths in the country, declined in last one year.
ORS coverage dropped by 13 percentage points, exclusive breastfeeding by 10 percentage points, whereas access to pneumonia care decreased by four percentage points in one year.
With a total of 2,60,990 deaths of children under 5 years of age due to pneumonia and diarrhoea, India continues to top the list of 15 countries with the most casualties. While 1,58,176 children under 5 died of pneumonia, diarrhoea accounted for 1,02,813 deaths in 2016.
Female children in India lose out on vaccination despite improvements in full immunization coverage. Only 78 females were fully immunized for every 100 males fully immunized in poor areas of Delhi. Addressing these disparities can help reduce pneumonia and diarrhoea burden.
Globally, pneumonia and diarrhoea cause 1.36 million deaths in children in the same category in 2016. Over two thirds of the global burden of pneumonia and diarrhoea mortality occurs in just 15 countries. Despite significant reductions of disease in recent years with improvements in access to and use of health interventions, nearly half a million pneumonia and diarrhoea deaths still occurred in just India and Nigeria.
The report lists exclusive breastfeeding, vaccination, access to care and use of antibiotics, oral re-hydration solution, and zinc supplementation as key interventions to help prevent and treat pneumonia and diarrhoea.
3. Consider the following statements about Trans Fats: 1. They are the healthiest type of fats. 2. They are produced when oil is hydrogenated. Which of the above statements is/are correct:
    A. 1 Only B. 2 Only C. Both 1 and 2 D. None

Answer - Option B
Explanation:
Trans fats:
Trans fatty acids (TFAs) or Trans fats are the most harmful type of fats which can have much more adverse effects on our body than any other dietary constituent. These fats are largely produced artificially but a small amount also occurs naturally. Thus in our diet, these may be present as Artificial TFAs and/ or Natural TFAs.
Artificial TFAs are formed when hydrogen is made to react with the oil to produce fats resembling pure ghee/butter.
In our diet the major sources of artificial TFAs are the partially hydrogenated vegetable oils (PHVO)/vanaspati/ margarine while the natural TFAs are present in meats and dairy products, though in small amounts.
TFAs pose a higher risk of heart disease than saturated fats. While saturated fats raise total cholesterol levels, TFAs not only raise total cholesterol levels but also reduce the good cholesterol (HDL), which helps to protect us against heart disease.
Trans fats consumption increases the risk of developing heart disease and stroke.
It is also associated with a higher risk of developing obesity, type 2 diabetes, heart disease, metabolic syndrome, insulin resistance, infertility, certain types of cancers and can also lead to compromised fetal development causing harm to the yet to be born baby.
Permissible limit:
WHO recommends that trans fat intake be limited to less than 1% of total energy intake and has called for the total elimination of TFAs in global food supply by 2023.
FSSAI has proposed to limit TFA limit in foods to 2% and eliminate trans fats from foods by 2022.
4. Global Education Monitoring Report is released by:
    A. United Nations Development Programme B. UNESCO C. World Bank D. None of the Above

Answer - Option B
Explanation:
UNESCO global education monitoring report 2019 has been released. Highlights of the report: Literacy levels in rural households of India dip with seasonal migration:
In India, 10.7 million children aged 6 to 14 lived in rural households with a seasonal migrant in 2013. About 28% of youth aged 15 to 19 in these households were illiterate or had not completed primary school, compared to 18% of the cohort overall.
About 80% of seasonal migrant children in seven cities lacked access to education near work sites, and 40% are likely to end up in work rather than education, experiencing abuse and exploitation.
Inter-State Migration:
Inter-State migration rates have doubled between 2001 and 2011. An estimated 9 million migrated between States annually from 2011 to 2016. The report also warns of the negative impact on education for children who are left behind as their parents migrate.
The worst hit- Construction labors:
The construction sector absorbs the majority of short-term migrants. A survey in Punjab state of 3,000 brick kiln workers in 2015-16 found that 60% were inter-State migrants. Between 65% and 80% of all children aged five to 14 living at the kilns worked there seven to nine hours per day. About 77% of kiln workers reported lack of access to early childhood or primary education for their children.
What has been done in this regard?
India has taken steps to address the issue.
The Right to Education Act in 2009 made it mandatory for local authorities to admit migrant children.
National-level guidelines were issued, allowing for flexible admission of children, providing transport and volunteers to support with mobile education.
The policies were attempted to create seasonal hostels and aiming to improve coordination between sending and receiving districts and states.
Some State governments have also taken steps for migrant children’s education. Pending Issues:
TMost interventions are focused on keeping children in home communities instead of actively addressing the challenges faced by those who are already on the move.
5. 25. Institution Innovation Councils are being set up in Higher Educational Institutions to foster and sustain a culture of Innovation. The initiative is under the overall management of:
    A. Ministry of Electronics and Information Technology. B. Ministry of Human Resources Development. C. Ministry of Science and Technology. D. Ministry of Earth Sciences.

Answer - Option B
Explanation:
Aim of Institution’s Innovation Councils (IICs):
To encourage, inspire and nurture young students by exposing them to new ideas and processes resulting in innovative activities in their formative years.
Major Focus of IIC –
To create a vibrant local innovation ecosystem.
Start-up supporting Mechanism in HEIs.
Prepare institute for Atal Ranking of Institutions on Innovation Achievements Framework.
Establish Function Ecosystem for Scouting Ideas and Pre-incubation of Ideas.
Develop better Cognitive Ability for Technology Students.
Why HEIs should join IICs Network?
No major capital investment required for establishing IIC as it will make use of existing local ecosystem.
Students/Faculty associated with IIC will get exclusive opportunity to participate in various Innovation related initiatives and competitions organized by MHRD.
Win exciting Prizes/Certificates every month.
Meet/Interact with renowned Business Leaders and top-notch academicians.
Opportunity to nurture and prototype new ideas.
Mentoring by Industry Professionals.
Experimentation with new/latest technologies.
Visit new places and learn a new culture.
6. Consider the following statements about Himalayan State Regional Council: 1. The aim of the council will be to ensure sustainable development in Himalayan Region. 2. It was constituted by Planning Commission in 2010. Which of the above statements is/are correct:
    A. 1 Only B. 2 Only C. Both 1 and 2 D. None

Answer - Option A
Explanation:
NITI Aayog has constituted the ‘Himalayan State Regional Council’ to ensure sustainable development of the Indian Himalayan region.
About the Council:
Composition: The Himalayan State Regional Council will be chaired by the Dr VK Saraswat, Member, NITI Aayog and will consist of the Chief Secretaries of the Himalayan States as well as the Secretaries of key Central Ministries, senior officers of NITI Aayog as well as special invitees.
Objective: The Council has been constituted to review and implement identified action points based on the Reports of five Working Groups, which were established along thematic areas to prepare a roadmap for action.
Functions: The Himalayan States Regional Council will be the nodal agency for the Sustainable development in the Himalayan Region which consists of the twelve States namely Jammu &Kashmir, Uttarakhand, Himachal Pradesh, Arunachal Pradesh, Manipur, Meghalaya, Mizoram, Nagaland, Sikkim, Tripura, two districts of Assam namely Dima Hasao and KarbiAnglong and Darjeeling and Kalimpong in West Bengal.
7. Consider the following statements about Aadi Mahotsav: 1. It is organized every year by Ministry of Social Justice and Empowerment. 2. The purpose of the festival is to celebrate the spirit to Tribal Art and Craft and its culture. Which of the above statements is/are correct:
    A. 1 Only B. 2 Only C. Both 1 and 2 D. None

Answer - Option B
Explanation:
“Aadi Mahotsav” a National Tribal Festival is being organized in New Delhi by the Ministry of Tribal Affairs and TRIFED to celebrate, cherish and promote the spirit of tribal craft, culture, cuisine and commerce.
The theme of the festival is: “A Celebration of the Spirit of Tribal Culture, Craft, Cuisine and Commerce”.
About Aadi Mahotsav:
The Mahotsav will comprise of display and sale of items of tribal art and craft, tribal medicine & healers, tribal cuisine and display of tribal folk performance, in which tribal artisans, chefs, folk dancers/musicians from 23 States of the country shall participate and provide glimpse of their rich traditional culture.
The festival will feature exhibition-cum-sale of tribal handicrafts, art, paintings, fabric, jewellery and much more through 100 stalls. Over 200 tribal artisans and artists from different States creating a Mini-India will be participating in the festival.
8. Consider the following statements about Maternity Benefit Act 2017: 1. It is applicable to establishments employing 10 or more persons. 2. The period of maternity leave has been increased from 12 to 26 weeks under the amended provisions. 3. Crèche facility has to be provided by establishments employing more than 30 employees. Which of the above statements is/are correct:
    A. 1 and 2 Only B. 1 and 3 Only C. 1 Only D. All are Correct.

Answer - Option A
Explanation:
The Maternity Benefit Act:
The Maternity Benefit Act, 1961, applies to establishments employing 10 or more than 10 persons in factories, mines, plantation, shops & establishments and other entities.
The main purpose of this Act is to regulate the employment of women in certain establishments for certain period before and after child birth and to provide maternity benefit and certain other benefits. The Act was amended through the Maternity Benefit (Amendment) Act, 2017.
The amendment has brought in major changes to the law relating to maternity benefits. These are:
It extends the period of maternity benefit from 12 weeks to 26 weeks of which not more than eight weeks can precede the date of the expected delivery. This exceeds the International Labour Organisation’s minimum standard of 14 weeks and is a positive development. However, a woman who has two or more surviving children will be entitled to 12 weeks of which not more than six weeks can precede the date of the expected delivery.
Women who legally adopt a child below the age of three months or a “commissioning mother” will be entitled to maternity benefit for 12 weeks from the date on which the child is handed over to her. A commissioning mother is defined as a biological mother who uses her egg to create an embryo implanted in another woman.
It gives discretion to employers to allow women to work from home after the period of maternity benefit on mutually agreeable conditions. This would apply if the nature of work assigned to the woman permits her to work from home
It requires establishments having 50 or more employees to have a crèche facility, either separately or along with common facilities. Further, employers should allow the woman to visit the crèche four times a day, which “shall also include the interval for rest allowed to her.”
It introduces a provision which requires every establishment to intimate a woman at the time of her appointment of the maternity benefits available to her. Such communication must be in writing and electronically.
9. Recently, Global IT Challenge for Youth with Disabilities was organized in New Delhi by:
    A. Ministry of Youth Affairs. B. Ministry of Health and Family Welfare C. Ministry of Electronics and Information Technology. D. Ministry of Social Justice and Empowerment

Answer - Option D
Explanation:
The Global IT Challenge for Youth with Disabilities, 2018, being organised by the Department of Empowerment of Persons with Disabilities (DEPwD), Ministry of Social Justice and Empowerment in association with Rehabilitation International Korea and their associated partner LG Electronics, is being held in New Delhi.
About the Global ICT Challenge for Youth with Disabilities:
The Global ICT Challenge for Youth with Disabilities is a capacity building project that helps youth with disabilities to overcome their limitations and challenge themselves for a better future by providing them with access to ICT and related experiences, improving their ability to leverage information and social participation while setting ICT agendas for participant countries related to disabilities and boosting international cooperation and exchange.
The objective of the event is to leverage IT skills among youth with disabilities and also to spread awareness about the application of Information and Computer Technology (ICT) in enhancing the quality of life of persons with disabilities especially in Asia-Pacific region.
10. Government of India has set an ambitious target to eliminate TB and Malaria by:
    A. 2025 and 2028 B. 2025 and 2030 C. 2022 and 2025 D. 2020 and 2025

Answer - Option B
Explanation:
Government has set a target of eliminating TB by 2025 and malaria by 2030.
India contributes to 27% of the global TB burden and 68% of all malaria cases in the Southeast Asia region.
1. [latex]{7}^{th}[/latex] International Tourism Mart was recently organized by Ministry of Tourism in which state:
    A. Agartla, Tripura. B. Dehradun, Uttrakhand C. Chennai, Tamil Nadu D. Gandhinagar, Gujarat

Answer - Option A
Explanation:
The Ministry of Tourism, Government of India, in association with the Department of Tourism, Government of Tripura and the North Eastern States is organizing the 7th “International Tourism Mart” in Agartala, Tripura.
This is the 7th edition of the International Tourism Mart, an annual event organised in the North Eastern region with the objective of highlighting the tourism potential of the region in the domestic and international markets.
It brings together the tourism business fraternity and entrepreneurs from the eight North Eastern States.
2. Consider the following statements about Prevention of Corruption Act: 1. The act provides for minimum imprisonment of 3 Years extendable up to 7 Years in cases of corruption. 2. It also criminalizes corporate corruption. Which of the above statements is/are correct:
    A. 1 Only B. 2 Only C. Both 1 and 2 D. None

Answer - Option C
Explanation:
Highlights of the Prevention of Corruption (Amendment) Bill, 2018:
Punishment for bribe-taking enhanced: Minimum punishment of 3 yrs, extendable up to 7 yrs with fine; from the earlier 6 months, with extension up to 3 yrs.
‘Undue Advantage’ expanded: The earlier limited definition of “undue advantage” expanded to now include “anything other than legal remuneration”.
Gifts criminalized: Gifts received for established undue advantage/mala-fide motive are now considered an act of corruption.
Collusive bribe-givers criminalized: For the first time, the giving of bribe has now been made a direct offence on par with taking of bribe. At the same time, protection has been built-in against coercive bribery, as long as the victim comes forward within 7 days.
Corporate bribery criminalized: Superiors to be held if employee/agent has bribed with their approval, for advancement of the organisation’s interests.
Immediate forfeiture: Law enforcement empowered for immediate attachment & forfeiture of illegal property of a public servant, invoking provisions of the Prevention of Money Laundering Act (PMLA).
Timely trial mandated: To conclude the investigation and trial within 2 yrs, extendable up to 4 yrs.
3. Proposed Regional Coalition “Quad” includes:
    A. India, Japan, Australia and U.S B. India, Indonesia, Malaysia and Japan. C. India, Japan, Australia and Myanmar. D. India, Bangladesh, Japan and Australia

Answer - Option A
Explanation:
Regional coalition known as the ‘Quad’, the quadrilateral formation includes Japan, India, United States and Australia.
All four nations find a common ground of being the democratic nations and common interests of unhindered maritime trade and security.
The idea was first mooted by Japanese Prime Minister Shinzo Abe in 2007. However, the idea couldn’t move ahead with Australia pulling out of it.
4. Consider the following statements regarding the Leadership for Academicians Programme (LEAP): 1. LEAP is a youth leadership development training programme of the Ministry of Youth Affairs and Sports. 2. Both domestic and foreign training are a part of the initiative. 3. The implementation of LEAP Programme will be through National institutional ranking framework’s top ranked Indian Institutions. Which of the above statements are correct?
    A. 1 and 2 only B. 2 and 3 only C. 1 and 3 only D. 1, 2 and 3

Answer - Option B
Explanation:
Leadership for Academicians Programme (LEAP)
Statement 1 is incorrect: Leadership for Academicians Programme (LEAP) is an initiative launched by, Ministry of Human Resource Development for higher education faculty.
Statement 2 is correct: It is a three weeks Flagship leadership development training programme (2 weeks domestic and one week foreign training) for second level academic functionaries in public funded higher education institutions.
The main objective is to prepare second tier academic heads who are potentially likely to assume leadership roles in the future. The programme would provide senior faculty, with high academic credentials, the required leadership and managerial skills including skills of problem-solving, handling stress, team building work, conflict management, developing communication skills, understanding and coping with the complexity and challenges of governance in Higher educational Institutes, financial & general administration.
Statement 3 is correct: The implementation of LEAP Programme will be through 15 National Institutional Ranking Framework (NIRF) top ranked Indian Institutions.
5. East Asia Summit 2018 was held in:
    A. New Delhi B. Cambodia C. Vietnam D. Singapore

Answer - Option D
Explanation:
[latex]{13}^{th}[/latex] East Asia Summit is being held in Singapore.
India has been participating in the EAS since its very inception in 2005.
6. Recently, which country launched the world’s first Sovereign Blue Bond?
    A. Republic of Seychelles B. Maldives C. Mauritius D. Philippines

Answer - Option A
Explanation:
The Republic of Seychelles launched the world’s first Sovereign Blue Bond, a financial instrument designed to support sustainable marine and fisheries projects.
7. Consider the following statements about World Tourism Organisation: 1. It is a U.N body responsible for the promotion of responsible, sustainable and universally accessible tourism. 2. Its recent session took place in Manama, Bahrain. Which of the above statements is/are correct:
    A. 1 Only B. 2 Only C. Both 1 and 2 D. None

Answer - Option C
Explanation:
[latex]{109}^{th}[/latex] session of United Nations World Tourism Organization (UNWTO) Executive Council is being held in Manama, Bahrain.
The Executive Council will discuss several topics on the agenda that are related to the development of global Tourism sector.
About UNWTO:
What is it?
The World Tourism Organization (UNWTO) is the United Nations agency responsible for the promotion of responsible, sustainable and universally accessible tourism.
Members:
UNWTO’s membership includes 158 countries, 6 Associate Members and over 500 Affiliate Members representing the private sector, educational institutions, tourism associations and local tourism authorities.
8. Which of the following statement clearly explains the objective of Mission “Raksha Gyan Shakti”?
    A. An annual event to engage public and private defence companies for greater collaboration. B. An annual event to encourage implementation of viable defence technologies in civilian applications. C. An event under Make-in-India to encourage Transfer-of-technology agreement between Indian and Foreign defence enterprises. D. To encourage defence Intellectual Property Rights in indigenous defence industry.

Answer - Option D
Explanation:
An IP Facilitation Cell was established in April this year which has worked tirelessly to achieve ambitious targets of training 10,000 personnel of OFB and DPSUs on IPR and to facilitate fi ling of at least 1,000 new IPR applications.
The Directorate General of Quality Assurance (DGQA) has been entrusted with the responsibility of coordinating and implementing the programme.
9. Consider the following statements about UN Habitat: 1. It is a U.N agency that focuses Human Settlements and Sustainable Urban Development. 2. The agency is a member of UN Development Group. Which of the above statements is/are correct:
    A. 1 Only B. 2 Only C. Both 1 and 2 D. None

Answer - Option C
Explanation:
About UN Habitat:
The United Nations Human Settlements Programme (UN–Habitat) is the United Nations agency for human settlements and sustainable urban development.
It was established in 1978 as an outcome of the First UN Conference on Human Settlements and Sustainable Urban Development (Habitat I) held in Vancouver, Canada in 1976.
It is mandated by the United Nations General Assembly to promote socially and environmentally sustainable towns and cities with the goal of providing adequate shelter for all. It is a member of the United Nations Development Group.
The mandate of UN-Habitat derives from the Habitat Agenda, adopted by the United Nations Conference on Human Settlements (Habitat II) in Istanbul, Turkey, in 1996.
The twin goals of the Habitat Agenda are adequate shelter for all and the development of sustainable human settlements in an urbanizing world.
10. Consider the following statements about Society for Worldwide Inter bank Financial Telecommunication: 1. It is a mechanism developed to facilitate inter bank Telecommunication. 2. It also facilitates fund transfers. Which of the above statements is/are correct:
    A. 1 Only B. 2 Only C. Both 1 and 2 D. None

Answer - Option A
Explanation:
What is SWIFT?
The SWIFT is a global member-owned cooperative that is headquartered in Brussels, Belgium.
It was founded in 1973 by a group of 239 banks from 15 countries which formed a co-operative utility to develop a secure electronic messaging service and common standards to facilitate cross-border payments.
It carries an average of approximately 26 million financial messages each day. In order to use its messaging services, customers need to connect to the SWIFT environment.
Functions:
SWIFT does not facilitate funds transfer: rather, it sends payment orders, which must be settled by correspondent accounts that the institutions have with each other.
The SWIFT is a secure financial message carrier — in other words, it transports messages from one bank to its intended bank recipient
Its core role is to provide a secure transmission channel so that Bank A knows that its message to Bank B goes to Bank B and no one else. Bank B, in turn, knows that Bank A, and no one other than Bank A, sent, read or altered the message en route. Banks, of course, need to have checks in place before actually sending messages.
1. Consider the following statements: 1. SAARC is a multi-lateral grouping operating in South Asia. 2. It is headquartered in Kathmandu. 3. It also has SAARC Development Fund based in New Delhi. Which of the above statements is/are correct:
    A. 1 and 3 Only B. 1 and 2 Only C. 1, 2 and 3 D. 2 and 3 Only

Answer - Option B
Explanation:
The South Asian Association for Regional Cooperation (SAARC) was established with the signing of the SAARC Charter in Dhaka on 8 December 1985. The Secretariat of the Association was set up in Kathmandu on 17 January 1987.
SAARC comprises of eight Member States: Afghanistan, Bangladesh, Bhutan, India, Maldives, Nepal, Pakistan and Sri Lanka.
Important objectives of the Association as outlined in the SAARC Charter are: to promote the welfare of the peoples of South Asia and to improve their quality of life; to accelerate economic growth, social progress and cultural development in the region and to provide all individuals the opportunity to live in dignity and to realize their full potentials; to promote and strengthen collective self-reliance among the countries of South Asia.
2. Consider following statements about International Telecommunication Union: 1. It is a UN body. 2. Its headquarters are based in Geneva. Switzerland. 3. It is the oldest existing international organization. Which of the above statements is/are correct:
    A. 1 and 2 Only B. 2 and 3 Only C. 1 and 3 Only D. All are Correct.

Answer - Option D
Explanation:
About International Telecommunication Union (ITU):
The International Telecommunication Union (ITU) is an agency of the United Nations (UN) whose purpose is to coordinate telecommunication operations and services throughout the world.
Originally founded in 1865, as the International Telegraph Union, the ITU is the oldest existing international organization.
ITU headquarters are in Geneva, Switzerland.
The ITU consists of three sectors:
1. Radio communication (ITU-R) — ensures optimal, fair and rational use of the radio frequency (RF) spectrum.
2. Telecommunication Standardization (ITU-T) — formulates recommendations for standardizing telecommunication operations worldwide.
3. Telecommunication Development (ITU-D) — assists countries in developing and maintaining internal communication operation
3. With reference to World Food Programme (WFP), consider the following statements: 1. It is food assistance branch of U.N. 2. It is the world’s largest Humanitarian organization. Which of the above is/are correct?
    A. 1 only B. 2 only C. Both 1 and 2 D. None

Answer - Option C
Explanation:
Justification: World Food Programme is the UN agency focused on hunger alleviation and food security. Globally, it responds to emergencies making sure food reaches where it is needed, especially in times of civil strife and natural disasters.
It is the largest humanitarian organization addressing hunger and promoting food security.
The WFP strives to eradicate hunger and malnutrition, with the ultimate goal in mind of eliminating the need for food aid itself. It is a member of the United Nations Development Group and part of its Executive Committee.
Born in 1961, WFP pursues a vision of the world in which every man, woman and child has access at all times to the food needed for an active and healthy life. The WFP is governed by an Executive Board which consists of representatives from member states.
The WFP operations are funded by voluntary donations from world governments, corporations and private donors. WFP food aid is also directed to fight micro nutrient deficiencies, reduce child mortality, improve maternal health, and combat disease, including HIV and AIDS.
4. Consider the following statements regarding Advanced Motor Fuels (AMF) Technology Collaboration Programme: 1. India is a member-state of the AMF Technology Collaboration Programme. 2. It is an International Energy Agency’s(IEA) transportation related Technology Collaboration Programme. 3. AMF fulfils its mission through international cooperation of academia, industries, governmental institutions, and non-government organizations Which of the above statements is/are correct?
    A. 1 and 2 only B. 2 only C. 1 and 3 only D. All are Correct

Answer - Option D
Explanation:
The Union Cabinet has been apprised of India joining as Member of Advanced Motor Fuels Technology Collaboration Programme (AMF TCP) under International Energy Agency (IEA).
The primary goal of joining AMF TCP by India to bring down emissions and achieve higher fuel efficiency in transport sector
What is Advanced Motor Fuels Technology Collaboration Programme?
AMF TCP is an international platform under the framework of International Energy Agency (IEA) for co-operation among countries to promote cleaner and more energy efficient fuels & vehicle technologies.
The activities of AMF TCP are deployment and dissemination of Advanced Motor Fuels. It looks upon the transport fuel issues in a systemic way taking into account the production, distribution and end use related aspects.
AMF TCP also provides an opportunity for fuel analysis, identifying new/ alternate fuels for deployment in transport sector and allied R&D activities for reduction in emissions in fuel intensive sectors.
5. Consider the following statements about G-20: 1. Countries from all the continents are represented in the Grouping. 2. India will host the summit in 2022. 3. The 2018 summit was the first ever summit in the South American continent. 4. The secretariat of G-20 is located in Washington, USA. Which of the given statements are correct?
    A. 2 and 3 only B. 2, 3 and 4 only C. 1, 2 and 3 only D. 1, 2, 3 and 4

Answer - Option A
Explanation:
Statement 3 is correct: 2018 G20 Buenos Aires summit, was the thirteenth meeting of Group of Twenty (G20), which was held on 30 November and 1 December 2018 in the city of Buenos Aires, Argentina. It was the first G20 summit to be hosted in South America.
Statement 1 is incorrect:
Countries from the African continent are not represented in the grouping
The G20 (or Group of Twenty) is an international forum for the governments and central bank governors from 19 countries and the European Union.
Founded in 1999 with the aim to discuss policy pertaining to the promotion of international financial stability, the G20 has expanded its agenda since 2008 and heads of government or heads of state, as well as finance ministers and foreign ministers, have periodically conferred at summits ever since. It seeks to address issues that go beyond the responsibilities of any one organization.
Statement 4 is incorrect: It’s a forum to highlight and discuss issues affecting largest group of countries (economically/politically) Hence it doesn’t have a secretariat.
6. Consider the following statements about Regional Comprehensive Economic Partnership: 1. It is a partnership proposed between ASEAN Nations including 6 other Nations with which ASEAN has Free Trade Agreement. 2. The other six nations are India, China, South Korea, Japan, Australia and New Zealand. Which of the above statements is/are correct:
    A. 1 Only B. 2 Only C. Both 1 and 2 D. None

Answer - Option C
Explanation:
10 ASEAN countries and six ASEAN FTA partners namely, India, China, Japan, Korea, Australia and New Zealand are members of the proposed RCEP.
ASEAN countries include Myanmar, Thailand, Cambodia, Vietnam, Laos, Singapore, Malaysia, Indonesia, Philippines and Brunei.
7. Consider the following statements about Asia Pacific Economic Cooperation: 1. It is a regional economic forum established in 1989 to leverage growing interdependence of Asia-Pacific. 2. APEC 2018 Summit held in Papua New Guinea. Which of the above statements is/are correct:
    A. 1 Only B. 2 Only C. Both 1 and 2 D. None

Answer - Option C
Explanation:
The 2018 APEC summit was held recently held in Papua New Guinea.
Outcome of the summit:
It ended with no joint statement from the leaders – a first in APEC history – and with the fight for dominance in the Pacific region between Australia, the US and Japan on one side and China on the other, coming out into the open.
APEC:
The Asia-Pacific Economic Cooperation (APEC) is a regional economic forum established in 1989 to leverage the growing interdependence of the Asia-Pacific. APEC has 21 members.
Aim: To create greater prosperity for the people of the region by promoting balanced, inclusive, sustainable, innovative and secure growth and by accelerating regional economic integration
8. Consider the following statements about World Customs Organisation: 1. The membership of the Organisation is divided into six regions. 2. Each Region is represented by regionally elected Vice-Chairperson. 3. It is headquartered in Brussels, Belgium. Which of the above statements is/are correct:
    A. 1 and 2 Only B. 2 and 3 Only C. 1 and 3 Only D. All are correct

Answer - Option D
Explanation:
What is it? WCO is independent intergovernmental body whose mission is to enhance effectiveness and efficiency of Customs administrations. It was established in 1952 as Customs Co-operation Council (CCC). It is the only international organization with competence in Customs matters and is considered as voice of international Customs community.
India has become the Vice-Chair (Regional Head) of the Asia Pacific Region of World Customs Organisation (WCO) for a period of two years, from July, 2018 to June, 2020.
The WCO has divided its Membership into six Regions. Each of the six Regions is represented by a regionally elected Vice Chairperson to the WCO Council.
9. Recently, a country was given humanitarian assistance in the form of health and nutritional aid by UN Central Emergency Response Fund. The country is:
    A. Ecuador B. Rwanda C. Pakistan D. Venezuela

Answer - Option D
Explanation:
The United Nations has announced $9.2 million in health and nutritional aid for crisis-stricken Venezuela, where hunger and preventable disease are soaring amid the collapse of the country’s socialist economic system.
About the UN Central Emergency Response Fund: It is a humanitarian fund established by the United Nations General Assembly on December 15, 2005 and launched in March 2006.
With CERF’s objectives to 1) promote early action and response to reduce loss of life; 2) enhance response to time-critical requirements; and 3) strengthen core elements of humanitarian response in underfunded crises, CERF seeks to enable more timely and reliable humanitarian assistance to those affected by natural disasters and armed conflicts.
The fund is replenished annually through contributions from governments, the private sector, foundations and individuals.
Allocations:
The CERF grant element is divided into two windows:
1. Rapid Responses (approximately two thirds of the grant element) The Rapid Response window provides funds intended to mitigate the unevenness and delays of the voluntary contribution system by providing seed money for life-saving, humanitarian activities in the initial days and weeks of a sudden onset crisis or a deterioration in an ongoing situation.
The maximum amount applied to a crisis in a given year typically does not exceed $30 million, although higher allocations can be made in exceptional circumstances.
2. Underfunded Emergencies (approximately one third of the grant element).
The Underfunded Emergencies window supports countries that are significantly challenged by “forgotten” emergencies.
10. Reggae, a genre of music, has recently been granted “intangible cultural heritage” status by UNESCO. It is traced to which among the following country
    A. Jamaica B. Kenya C. Venezuela D. Argentina

Answer - Option A
Explanation:
United Nations Educational, Scientific and Cultural Organization, has added reggae to its list of cultural institutions worthy of protection and preservation.
Background:
Each year, UNESCO adds to its Representative List of the Intangible Cultural Heritage of Humanity, and Jamaica submitted reggae for consideration earlier this year. The genre now joins a list of over 300 cultural traditions, including numerous musical ones such as Dominican merengue, Slovakian bagpipe music and Vietnamese xoan singing.
About Reggae: It is Jamaican music originating from marginalized groups, mainly in Western Kingston. It was started out as voice of marginalized. But now it is played and embraced by wide cross-section of society, including various genders, ethnic and religious groups.
About UNESCO’s List of Intangible Cultural Heritage: This coveted list is made up of those intangible heritage elements that help demonstrate diversity of cultural heritage and raise awareness about its importance. The list was established in 2008 when Convention for Safeguarding of the Intangible Cultural Heritage came into effect. It has two parts viz. Representative List of Intangible Cultural Heritage of Humanity and List of Intangible Cultural Heritage in Need of urgent safeguarding
From India the Intangible Cultural Heritages added into this list include:
Tradition of Vedic chanting
Ramlila, the traditional performance of the Ramayana
Kutiyattam, Sanskrit theatre
Ramman, religious festival and ritual theatre of the Garhwal Himalayas.
Mudiyettu, ritual theatre and dance drama of Kerala
Kalbelia folk songs and dances of Rajasthan
Chhau dance
Buddhist chanting of Ladakh: recitation of sacred Buddhist texts in the trans-Himalayan Ladakh region, Jammu and Kashmir.
Sankirtana, ritual singing, drumming and dancing of Manipur
Traditional brass and copper craft of utensil making among the Thatheras of Jandiala Guru, Punjab
Yoga
Nawrouz
Kumbh Mela
1. A Public Credit Registry will 1. Provide information on borrowings only from scheduled commercial banks and NBFCs. 2. Help improve India’s ranking in the World Bank’s Ease of Doing Business. Select the correct answer using the code given below:
    A. 1 only B. 2 only C. Both 1 and 2 D. Neither 1 nor 2

Answer - Option B
Explanation:
A comprehensive credit information repository covering all types of credit facilities (funded and non-funded) extended by all credit institutions – Commercial Banks, Cooperative Banks, NBFCs, MFIs – and also covering borrowings from other sources, including external commercial borrowings and borrowing from market, is essential to ascertain the total indebtedness of a legal or natural person.
With technology acting as an enabler, this repository can make near real time monitoring of credit risk possible and also address legitimate privacy concerns of the borrowers by making all access to a borrower’s information contained in the repository dependent on the borrower’s consent.
With a PCR in place, and with full coverage of credit market ensured by mandatory reporting, the ease in getting credit and in turn the ranking of India in the World Bank’s ease of doing business index would also likely to be improved.
The Reserve Bank of India (RBI) had formed a high-level task force on public credit registry (PCR) for India. The task force was chaired by Y M Deosthalee.
The task force has suggested the registry should capture all loan information and borrowers be able to access their own history. Data is to be made available to stakeholders such as banks, on a need-to-know basis. Data privacy will be protected.
2. Consider the following statements in the context of Payments Regulatory Board: 1. It will replace the existing Board for Regulation and Supervision of Payment and Settlement Systems. 2. RBI Governor will be the ex-officio chairperson of the board. 3. Ratan Watal Committee for Digital Payments recommended the establishment of Payment Regulatory Board. Which of the above statements are correct?
    A. 1 and 2 only B. 2 and 3 only C. 1 and 3 only D. 1, 2 and 3

Answer - Option D
Explanation:
Payments Regulatory Board
The Union Government in Budget 2017-18 has proposed creation of a six member Payments Regulatory Board in the Reserve Bank of India (RBI) as part of bringing about structural reforms in the payment eco-system.
Statement 1 is correct: It designates RBI as the nodal authority for the regulation and supervision of payment systems. It will replace the existing Board for Regulation and Supervision of Payment and Settlement Systems. For the purpose, necessary amendment are proposed in the Finance Bill 2017 to amend the Payment and Settlement Systems Act, 2007.
Statement 2 is correct: Composition: RBI Governor will be the ex officio Chairperson, Deputy Governor who is in—charge of the Payment and Settlement Systems will be member, and one officer of RBI will be nominated by its Central Board will be its members.
Statement 3 is correct: Ratan Watal Committee for Digital Payments the 11- member committee was notified in September 2016 by the Union Finance Ministry to review existing payment systems in country and recommend appropriate measures for encouraging Digital Payments. It recommended the establishment of Payment Regulatory Board.
3. Consider the following statements about Prompt Corrective Action: 1. It is a plan to help banks come out of stressed assets situation. 2. It is initiated by RBI. Which of the above statements is/are correct:
    A. 1 only B. 2 only C. Both 1 and 2 D. Neither 1 nor 2

Answer - Option C
Explanation:
PCA norms allow the regulator to place certain restrictions such as halting branch expansion and stopping dividend payment. It can even cap a bank’s lending limit to one entity or sector. Other restructuring operations and activation of recovery plan.
Banks’ promoters can be asked to bring in new management, too. The RBI can also supersede the bank’s board, under PCA.
The PCA is invoked when certain risk thresholds are breached. There are three risk thresholds which are based on certain levels of asset quality, profitability, capital and the like.
The third such threshold, which is maximum tolerance limit, sets net NPA at over 12% and negative return on assets for four consecutive years.
The 11 banks under the PCA are: Allahabad Bank, United Bank of India, Corporation Bank, IDBI Bank, UCO Bank, Bank of India, Central Bank of India, Indian Overseas Bank, Oriental Bank of Commerce, Dena Bank and Bank of Maharashtra. These banks together control over 20 percent of the credit market.
4. Consider the following statements on Pradhan Mantri Mudra Yojana: 1. The scheme has a corpus of Rs20,000 Crore. 2. The loans given under the Scheme are of three types i.e. Shishu, Kishore and Tarun. Which of the above statements is/are correct:
    A. 1 only B. 2 only C. Both 1 and 2 D. Neither 1 nor 2

Answer - Option C
Explanation:
The PMMY Scheme was launched in April, 2015. The scheme’s objective is to refinance collateral-free loans given by the lenders to small borrowers.
The scheme, which has a corpus of Rs 20,000 crore, can lend between Rs 50,000 and Rs 10 lakh to small entrepreneurs.
Banks and MFIs can draw refinance under the MUDRA Scheme after becoming member-lending institutions of MUDRA.
Mudra Loans are available for non-agricultural activities upto Rs. 10 lakh and activities allied to agriculture such as Dairy, Poultry, Bee Keeping etc, are also covered.
There are three types of loans under PMMY:
1. Shishu (up to Rs.50,000).
2. Kishore (from Rs.50,001 to Rs.5 lakh).
3. Tarun (from Rs.500,001 to Rs.10,00,000).
Objectives of the scheme:
Fund the unfunded: Those who have a business plan to generate income from a non-farm activity like manufacturing, processing, trading or service sector but don’t have enough capital to invest can take loans up to Rs 10 lakh.
Micro finance institutions (MFI) monitoring and regulation: With the help of MUDRA bank, the network of micro finance institutions will be monitored. New registration will also be done.
Promote financial inclusion: With the aim to reach Last mile credit delivery to micro businesses taking help of technology solutions, it further adds to the vision of financial inclusion.
Reduce jobless economic growth: Providing micro enterprises with credit facility will help generate employment sources and an overall increase in GDP.
Integration of Informal economy into Formal sector: It will help India also grow its tax base as incomes from the informal sector are non-taxed.
5. Consider the following statements: 1. Ease of Doing Business Report is released annually by World Bank. 2. This year rank dropped to 77 among 190 Nations. Which of the above statements is/are correct:
    A. 1 only B. 2 only C. Both 1 and 2 D. Neither 1 nor 2

Answer - Option A
Explanation:
The World Bank has released its latest Doing Business Report (DBR, 2019).
The report ranks 190 economies based on how easy it is to do business there, taking into account trading regulations, property rights, contract enforcement, investment laws, the availability of credit and a number of other factors. The first report was published in 2003.
Performance of various countries:
The top five overall are: New Zealand, Singapore, Denmark, Hong Kong and Korea. The U.S. ranks 8th, down from 6th last year.
As per the latest report, Afghanistan had moved up the most, by 16 spots, from 183 th in last year’s ranking to 167th this year.
China and India — two economies with the largest populations — are among top 10 improves this year.
Afghanistan, Djibouti, Azerbaijan, Togo, Kenya, Côte d’Ivoire, Turkey and Rwanda rounded out the top 10 most improved list. In addition, Djibouti and India are the only economies to make the 10 top most improved list for the second consecutive year.
Performance of India:
India climbed 23 spots from a year ago to rank 77 out of 190 countries in the World Bank’s latest report on the ease of doing business.
It was also among the top 10 most improved economies along with countries such as China, Djibouti and Azerbaijan.
The ease of doing business in India improved notably after a series of reforms made it easier for companies to get construction permits, pay taxes and trade across borders.
6. Recently, International Labour Organisation released Global Wage Report. As per the report, following country has seen the highest Gender Wage Gap:
    A. Pakistan B. China C. India D. Brazil

Answer - Option C
Explanation:
The International Labour Organization (ILO) has released Global Wage Report 2018-19. As per the report, women are paid the most unequally in India, compared to men, when it comes to hourly wages for labour. On average, women are paid 34% less than men.
Highlights of the report:
The gender wage gap has remained unchanged at 20% from 2016 to 2017.
In advanced economies (G20), real wage growth declined from 0.9 per cent in 2016 to 0.4 per cent in 2017, meaning near stagnation. By contrast, in emerging economies and developing G20 countries, real wage growth dipped marginally from 4.9 per cent in 2016 and 4.3 per cent in 2017.
Inequality is higher in monthly wages, with a gap of 22 per cent. Overall, real wages grew just 1.8 per cent globally (136 countries) in 2017.
In most countries, women and men differ significantly in respect of working time – specifically, that part-time work is more prevalent among women than among men.
Way Forward:
The report advocated that emphasis needs to be placed on ensuring equal pay for women and men.
7. World Fisheries Day is celebrated on:
    A. 21 November B. 9 November C. 15 November D. 24 November

Answer - Option A
Explanation:
World Fisheries Day is celebrated on November 21st every year to highlight the importance of healthy oceans ecosystems and to ensure sustainable stocks of fisheries in the world.
8. Consider the following statements about India Meteorological Department: 1. Formed in 1875, it functions under Ministry of Earth Sciences. 2. It is headquartered in Chennai. 3. It also deals with seismology apart from meteorology. Which of the above statements is/are correct:
    A. 1 and 2 Only B. 1 and 3 Only C. 1 Only D. All are Correct

Answer - Option B
Explanation:
India Meteorological Department (IMD):
It is national meteorological service of the country and chief government agency dealing in everything related to meteorology, seismology and associated subjects.
It was formed in 1875. It functions under Ministry of Earth Sciences. It is headquartered in New Delhi.
Mandate:
Undertake meteorological observations and provide current information and forecasting information for most favorable operation of weather-dependent activities such as irrigation, agriculture, aviation, shipping etc.
Offer warning against severe weather phenomenon such as tropical cyclones, northwesters, dust storms, heat waves, cold waves, heavy rains, heavy snow, etc.
Provide met-related statistics needed for agriculture, industries, water resources management, oil exploration, and any other strategically important activities for the country.
Engage in research in meteorology and allied subjects.
Detect and locate earthquakes and evaluate seismicity in various parts of the country for developmental projects.
9. Consider the following statements about Sagar Vani: 1. It will help to serve the coastal community, especially the fishermen. 2. It is a software platform where various dissemination modes will be integrated on a single central server. 3. It is developed under the Ministry of Earth Science. Which of the above statements is/are correct?
    A. 1 only B. 1 and 2 only C. 1, 2 and 3 only D. 2 and 3 only

Answer - Option C
Explanation:
Sagar Vani
Statement 1 is correct: Sagar Vani is an integrated information dissemination system that will serve the coastal community, especially the fishermen community with the advisories and alerts towards their livelihood as well as their safety at Sea.
Statement 2 is correct: The ‘Sagar Vani’ is a software platform where various dissemination modes will be integrated on a single central server. The system also has facility to provide access to various stakeholders (NGOs, State Fishery Departments, Disaster Management Authorities, etc.) so that they too will be able to further disseminate these ocean information and alerts to the user community.
It seeks to target the reach of information to the 9.27 lakh people involved in actual fishing either full or part time. The system that can send information in regional languages through various platforms including radio, television, voice calling, text service, social media, mobile apps and email aims at timely dissemination of ocean information and advisory services that includes Potential Fishing Zone (PFZ) advisories, Ocean State Forecast (OSF), High Wave Alerts and Tsunami early warnings.
Statement 3 is correct: The advanced system has been developed by Indian National Centre for Ocean Information Services (INCOIS), under the Ministry of Earth Science, through its industry partner, Gaian Solutions Pvt. Ltd. It includes Multi Lingual SMS, Voice Call / Audio Advisory, Mobile Apps (User / Admin modules), Social Media (Facebook, Twitter, etc.), Email, GTS, Fax, Digital Display Boards, Radio / Television broadcast units, IVRS, Cloud Channels, etc.
10. Advent of which of the following technologies is often referred to as Industrial Revolution 4.0? 1. Cloud computing 2. Computers 3. Internet of Things 4. Telecommunication Select the correct answer using the codes given below
    A. 1 and 3 only B. 3 and 4 only C. 2 and 4 only D. All are Correct

Answer - Option A
Explanation:
First Industrial Revolution:
The First Industrial Revolution took place from the 18th to 19th centuries in Europe and America. It was a period when mostly agrarian, rural societies became industrial and urban. The iron and textile industries, along with the development of the steam engine, played central roles in the Industrial Revolution.
Second Industrial Revolution:
The Second Industrial Revolution took place between 1870 and 1914, just before World War I. It was a period of growth for pre-existing industries and expansion of new ones, such as steel, oil and electricity, and used electric power to create mass production. Major technological advances during this period included the telephone, light bulb, phonograph and the internal combustion engine.
Third Industrial Revolution:
The Third Industrial Revolution, or the Digital Revolution, refers to the advancement of technology from analog electronic and mechanical devices to the digital technology available today.
The era started during the 1980s and is ongoing. Advancements during the Third Industrial Revolution include the personal computer, the internet, and information and communications technology (ICT)
Fourth Industrial Revolution
Option 1 and 3 are correct: The Fourth Industrial Revolution builds on the Digital Revolution, representing new ways in which technology becomes embedded within societies and even the human body.
The Fourth Industrial Revolution is marked by emerging technology breakthroughs in a number of fields, including robotics, artificial intelligence, nanotechnology, quantum computing, Cloud Computing, biotechnology, The Internet of Things (IoT), 3D printing and autonomous vehicles.
1. With reference to Saura Jalnidhi Scheme, which of the following statements are correct? 1. It is an initiative to promote irrigation in the Saurashtra region of Gujrat. 2. 90% subsidy and solar pumps will be provided by the state government. 3. Farmer identity cards are an important component of the scheme. Select the correct answer using the code given below:
    A. 1 and 2 only B. 2 and 3 only C. 1 and 3 only D. 1, 2 and 3

Answer - Option B
Explanation -
Saura Jalnidhi Scheme
Statement 1 is incorrect: It is an initiative of the Odisha Government to encourage use of solar energy in irrigation by farmers.
Key features:
Statement 3 is correct: The beneficiary of this scheme will be farmers who have valid farmer identity cards and have minimum 0.5 acres of agricultural land.
Statement 2 is correct: Farmers will be given 90% subsidy and 5,000 solar pumps at subsidized rates.
In the first phase, this scheme will be available in those areas where electricity is not available for running the pump set.
Farmers will be provided with a well equipped solar pump irrigation system in convergence mode.
Benefits:
1.52 lakh human day employments annually,
Livelihood to around 5,000 families,
Reduced carbon footprints
Promote the use of green energy and Reduced Pollution.
2. Consider the following statements regarding the National Council on India’s Nutrition Challenges: 1. It is chaired by the Prime Minister 2. Its mandate includes maintaining coordination and review convergence among ministries. 3. The Council submits its report to the NITI Aayog every six months. Which of the above statements is/are correct?
    A. 1 and 2 only B. 2 only C. 1 and 3 only D. 1, 2 and 3

Answer - Option B
Explanation -
National Council on India’s Nutrition Challenges
Statement 1 is incorrect: The Council which has been set up under POSHAN Abhiyaan (Prime Minister’s Overarching Scheme for Holistic Nutrition-National Nutrition Mission) is the apex body to formulate overall policies, guide and monitor all nutrition based schemes under the Chairmanship of Vice-Chairman (not Prime Minister) NITI Aayog.
The mandate of the Council is:
To provide policy directions to address India’s Nutrition Challenges through coordinated inter-sectoral action.
Statement 2 is correct: To coordinate and review convergence among ministries.
To review programmes for nutrition on a quarterly basis.
3. Consider the following statements with regard to Asian Development Bank: 1. Established in 1966, it is headquartered in Philippines. 2. Its objective is to promote Economic Development in Asia and to free Asia Pacific from poverty. 3. Voting system in the bank is based on equal share i.e. One member, One vote system. Which of the above statements is/are correct:
    A. 1 and 3 Only B. 2 Only C. 1 and 2 Only D. All are Correct.

Answer - Option C
Explanation -
It is a regional development bank established on 22 August 1966 and is headquartered in Philippines.
It aims to facilitate economic development of countries in Asia.
It also aims for an Asia and Pacific free from poverty.
The bank admits the members of the United Nations Economic and Social Commission for Asia and the Pacific (UNESCAP, formerly known as the United Nations Economic Commission for Asia and the Far East) and non-regional developed countries.
ADB was modelled closely on the World Bank, and has a similar weighted voting system where votes are distributed in proportion with member’s capital subscriptions.
Japan holds the largest proportions of shares at 15.67%. The United States holds 15.56%, China holds 6.47%, India holds 6.36%, and Australia holds 5.81%.
It offers both Hard Loans and Soft loans.
The ADB offers “hard” loans from ordinary capital resources (OCR) on commercial terms, and the Asian Development Fund (ADF) affiliated with the ADB extends “soft” loans from special fund resources with concessional conditions.
4. Consider the following statements about Saubhagya Scheme: 1. Rural Electrification Corporation is the nodal agency for the scheme. 2. It aims to provide free electricity connections to BPL households. Which of the above statements is/are correct:
    A. 1 Only B. 2 Only C. Both 1 and 2 D. None

Answer - Option A
Explanation -
Pradhan Mantri Sahaj Bijli Har Ghar Yojana – ‘Saubhagya’ was launched in September, 2017.
Under Saubhagya free electricity connections to all households (both APL and poor families) in rural areas and poor families in urban areas.
Rural Electrification Corporation (REC) has been designated as nodal agency for the Saubhagya scheme.
The scheme aims to achieve universal household electrification in all parts of the country at a cost of Rs 16,320 crore, including Gross Budgetary Support of Rs 12,320 crore from the government.
Its objective is to provide access to electricity connections to all the remaining households in the country.
Scheme will provide subsidy on equipment such as transformers, wires and meters.
Ministry of Power would be the implementing authority.
Power connection will be provided in both rural and urban areas of the country.
The scheme was launched on the occasion of the birth centenary of Pandit Deen Dayal Upadhyaya
Award Scheme under Saubhagya:
An award scheme has been instituted with awards of more than Rs. 300 crore to be won by States/Discoms.
The first DISCOM/Power Departments who complete 100% household electrification will be felicitated with cash award of Rs. 50 Lakh for the employees and 100 crore grants for distribution infrastructure.
The Government is committed to ensure 24×7 access to electricity for all by 31st March 2019.
5. Consider following statements about Atal Tinkering Labs: 1. These labs are established across schools in India with a vision to Cultivate one Million children in India as Neoteric Innovators. 2. These labs are established under the scheme “Atal Innovation Mission” 3. AIM will provide grant-in-aid that includes a one-time establishment cost of Rs. 10 lakh and operational expenses of Rs. 10 lakh for a maximum period of 5 years to each ATL. Which of the above statements is/are correct:
    A. 1 Only B. 1, 2 and 3 Only C. 2 and 3 Only D. 1 and 2 Only

Answer - Option B
Explanation -
With a vision to ‘Cultivate one Million children in India as Neoteric Innovators’, Atal Innovation Mission is establishing Atal Tinkering Laboratories (ATLs) in schools across India.
Objective: The objective of this scheme is to foster curiosity, creativity and imagination in young minds; and inculcate skills such as design mindset, computational thinking, adaptive learning, physical computing etc.
Financial Support: AIM will provide grant-in-aid that includes a one-time establishment cost of Rs. 10 lakh and operational expenses of Rs. 10 lakh for a maximum period of 5 years to each ATL.
Eligibility:
Schools (minimum Grade VI – X) managed by Government, local body or private trusts/society can set up ATL.
6. Experimental Advanced Superconducting Tokamak (EAST) reactor is an artificial sun designed to replicate the process our natural Sun. It is based in:
    A. Japan B. North Korea C. South Korea D. China

Answer - Option D
Explanation -
China has announced that its Experimental Advanced Superconducting Tokamak (EAST) reactor — an “artificial sun” designed to replicate the process our natural Sun uses to generate energy — just hit a new temperature milestone: 100 million degrees Celsius (180 million degrees Fahrenheit).
For comparison, the core of our real Sun only reaches about 27 million degrees Fahrenheit — meaning the EAST reactor was, briefly, more than six times hotter than the closest star.
7. Consider the following statements about Stratospheric Aerosols: 1. They are Sulphur rich particles. 2. The layer in which they are found is called Junge layer. Which of the above statements is/are correct:
    A. 1 Only B. 2 Only C. Both 1 and 2 D. None

Answer - Option C
Explanation -
What are Stratospheric Sulphur Aerosols?
Stratospheric sulfur aerosols are sulfur-rich particles which exist in the stratosphere region of the Earth’s atmosphere. The layer of the atmosphere in which they exist is known as the Junge layer, or simply the stratospheric aerosol layer.
These particles consist of a mixture of sulfuric acid and water. They are created naturally, such as by photochemical decomposition of sulfur-containing gases, e.g. carbonyl sulfide.
Sulfur aerosols are common in the troposphere as a result of pollution with sulfur dioxide from burning coal, and from natural processes.
Volcanoes are a major source of particles in the stratosphere as the force of the volcanic eruption propels sulfur-containing gases into the stratosphere.
Scientists have found that spraying sun-dimming chemicals high above the earth to slow global warming could be remarkably inexpensive costing about $2.25 billion a year over a 15-year period.
This geo-engineering technique known as stratospheric aerosol injection (SAI) could limit rising temperatures that are causing climate change.
8. Consider the following statements about India Meteorological Department: 1. Formed in 1875, it functions under Ministry of Earth Sciences. 2. It is headquartered in Chennai. 3. It also deals with seismology apart from meteorology. Which of the above statements is/are correct:
    A. 1 and 2 Only B. 1 and 3 Only C. 1 Only D. All are Correct

Answer - Option B
Explanation - India Meteorological Department (IMD): It is national meteorological service of the country and chief government agency dealing in everything related to meteorology, seismology and associated subjects. It was formed in 1875. It functions under Ministry of Earth Sciences. It is headquartered in New Delhi.
Mandate:
Undertake meteorological observations and provide current information and forecasting information for most favorable operation of weather-dependent activities such as irrigation, agriculture, aviation, shipping etc. Offer warning against severe weather phenomenon such as tropical cyclones, northwesters, dust storms, heat waves, cold waves, heavy rains, heavy snow, etc. Provide met-related statistics needed for agriculture, industries, water resources management, oil exploration, and any other strategically important activities for the country. Engage in research in meteorology and allied subjects. Detect and locate earthquakes and evaluate seismicity in various parts of the country for developmental projects.
9. A Public Credit Registry will 1. Provide information on borrowings only from scheduled commercial banks and NBFCs. 2. Help improve India’s ranking in the World Bank’s Ease of Doing Business. Select the correct answer using the code given below:
    A. 1 only B. 2 only C. Both 1 and 2 D. Neither 1 nor 2

Answer - Option B
Explanation:
A comprehensive credit information repository covering all types of credit facilities (funded and non-funded) extended by all credit institutions – Commercial Banks, Cooperative Banks, NBFCs, MFIs – and also covering borrowings from other sources, including external commercial borrowings and borrowing from market, is essential to ascertain the total indebtedness of a legal or natural person.
With technology acting as an enabler, this repository can make near real time monitoring of credit risk possible and also address legitimate privacy concerns of the borrowers by making all access to a borrower’s information contained in the repository dependent on the borrower’s consent.
With a PCR in place, and with full coverage of credit market ensured by mandatory reporting, the ease in getting credit and in turn the ranking of India in the World Bank’s ease of doing business index would also likely to be improved.
The Reserve Bank of India (RBI) had formed a high-level task force on public credit registry (PCR) for India. The task force was chaired by Y M Deosthalee.
The task force has suggested the registry should capture all loan information and borrowers be able to access their own history. Data is to be made available to stakeholders such as banks, on a need-to-know basis. Data privacy will be protected.
10. Consider the following statements about Namami Gange Programme: 1. The programme’s purpose is to clean River Ganges in an effective and coordinated manner by involving all stakeholders. 2. For the purpose of Project implementation, a three tier structure has been created at Central, State and District level. Which of the above statements is/are correct:
    A. 1 Only B. 2 Only C. Both 1 and 2 D. None

Answer - Option C
Explanation:
Namami Gange programme was launched as a mission to achieve the target of cleaning river Ganga in an effective manner with the unceasing involvement of all stakeholders, especially five major Ganga basin States – Uttarakhand, Uttar Pradesh, Jharkhand, Bihar and West Bengal.
The programme envisages: River Surface Cleaning, Sewerage Treatment Infrastructure, River Front Development, Bio-Diversity, Afforestation and Public Awareness
The program would be implemented by the National Mission for Clean Ganga (NMCG), and its state counterpart organizations i.e., State Program Management Groups (SPMGs). In order to improve implementation, a three-tier mechanism has been proposed for project monitoring comprising of a) High level task force chaired by Cabinet Secretary assisted by NMCG at national level, b) State level committee chaired by Chief Secretary assisted by SPMG at state level and c) District level committee chaired by the District Magistrate.
The program emphasizes on improved coordination mechanisms between various Ministries/Agencies of Central and State governments.
1. India’s GSAT-11 satellite was recently launched from the Guiana Space Centre. Which of the following statements is/are correct about GSAT-11? 1. GSAT-11 satellite is the heaviest satellite developed by ISRO. 2. It is able to provide greater speed and capacity in connectivity. 3. It will help in implementation of Bharat Net project. Select the correct answer using the code given below
    A. 1 and 2 only B. 2 and 3 only C. 3 only D. 1, 2 and 3

Answer - Option D
Explanation:
GSAT-11 Satellite
Statement 1 is correct: GSAT-11 also called Big Bird has been indigenously developed by ISRO. It weight 5854 kg making it heaviest Indian-made equipment that has been put into the orbit.
Statement 2 is correct: In a first for a satellite built by ISRO, GSAT-11 will carry a next-generation I-6K bus (communication satellite hub) to provide services in two widely-used wavelengths for telecommunications: the Ku- and Kabands.
This makes GSAT-11 three to six times more powerful than any of ISRO’s (and India’s) satellite roster today. It will provide up to 14 Gigabit/s in both voice and video broadband services anywhere in the Indian mainland or islands over its 15-year lifespan, according to ISRO.
GSAT-11 will bring far greater speeds (16 Gbps of it, no less) and capacity to meet growing demand for mobile and internet in households, businesses, and public organizations.
Statement 3 is correct: Large parts of rural areas still remain untouched by the scope of commercial telecom today.
Under Digital India’s Bharat Net project GSAT-11 will boost access to voice and video streaming in most, if not all, of rural India.
2. Consider the following pairs of Missile System and the countries they are imported from: Imported from [latex]\rightarrow [/latex] Missile System 1. USA [latex]\rightarrow [/latex] NASAMS-II 2. Russia [latex]\rightarrow [/latex] Igla-S 3. Israel [latex]\rightarrow [/latex] Barak 4. Iran [latex]\rightarrow [/latex] Iron Dome
    A. 1 only B. 2 only C. Both 1 and 2 D. Neither 1 nor 2

Answer - Option A
Explanation:
The ministry of road transport and highways has approved the sale of quadricycles for personal use.
Quadricycle – a vehicle of the size of a 3-wheeler but with 4 tyres and fully covered like a car. It has an engine like that of a 3-wheeler. Ministry of Road Transport & Highways notified the insertion of
‘Quadricycle’ as a ‘non transport’ vehicle under the Motor Vehicles Act 1988.
Quadricycles were only allowed for transport usage under the Act, but now has been made usable for non-transport also (Personal use)
Intention behind this move:
The main aim is to give another mobility option, especially to the middle class who want to upgrade from two-wheeler.
Besides, a person cannot carry family members in a three-wheeler, therefore, quadricycles are being allowed in the private category. The move will help them to upgrade to four-wheeler and move around with family.
3. Which of the following are Mars missions? 1. Insight 2. EVE 3. Wall-E 4. Viking Select the correct answer using the code given below:
    A. 2, 3 and 4 only B. 2 and 3 only C. 3 and 4 only D. 1, 2, 3 and 4

Answer - Option C
Explanation:
Statement 1 is correct: NASA’s Interior Exploration using Seismic Investigations, Geodesy and Heat Transport (InSight) is a robot which touched down on the Red Planet after an almost seven-month, 300- million-mile (458-million-kilometer) journey from Earth.
Its two-year mission will be to study the deep interior of Mars to learn how all celestial bodies with rocky surfaces including Earth and the Moon, formed.
Statement 2 are correct: Two small satellites WALL-E and EVE were launched on the Atlas V rocket that launched just after the Insight mission. Similar in size to a briefcase or large cereal box, the satellites are on way to Mars, right behind InSight.
This is the fi rst time that any little cubeshaped satellites, Cube-Sats as they’re known, have been sent to deep space missions. The journey will span 6 ½ months and 485 million kilometres. Insight is latest mission to Mars since 2012 Curiosity mission.
Statement 4 is correct: Viking is the first US mission to land a space craft safely on mars launched by NASA.
4. Consider the following statements about Agni I Missile: 1. It is long range surface to surface ballistic missile with specialised Navigation System to guide it. 2. It is developed by DRDO as part Integrated Guided Missile Development Program. 3. It is a single stage missile powered by solid propellants. Which of the above statements is correct:
    A. 2 and 3 Only B. 1, 2 and 3 C. 1 and 3 Only D. 2 Only

Answer - Option A
Explanation:
Agni-I is a short-range ballistic missile developed by DRDO of India under the Integrated Guided Missile Development Program. It is a single-stage missile developed after the Kargil War to fill the gap between 250 km range of Prithvi-II and 2,500 km range of Agni-II.
Agni-I missile has a specialised navigation system which ensures it reaches the target with a high degree of accuracy and precision.
5. What is ASTROSAT?
    A. Remote Sensing Satellite B. Multi wavelength Space Observatory C. Satellite revolving around the Moon D. Satellite dedicated to clear Space Junk

Answer - Option B
Explanation:
ASTROSAT:
ASTROSAT is India’s first dedicated multi wavelength space observatory. This scientific satellite mission endeavours for a more detailed understanding of our universe.
ASTROSAT observes universe in the optical, Ultra-violet, low and high energy X-ray regions of the electromagnetic spectrum, whereas most other scientific satellites are capable of observing a narrow range of wavelength band.
ASTROSAT with a lift-off mass of about 1513 kg was launched into a 650 km orbit inclined at an angle of 6 degree to the equator by PSLV-C30. After injection into Orbit, the two solar panels of ASTROSAT were automatically deployed in quick succession. The spacecraft control centre at Mission Operations Complex (MOX) of ISRO Telemetry, Tracking and Command Network (ISTRAC) at Bangalore manages the satellite during its mission life.
6. Consider the following statements about DAWN Mission, which was recently in news: 1. It was jointly launched by European and Russian space agencies. 2. It was sent to study the proto planets of the asteroid belt. Which of the above statements is/are INCORRECT?
    A. 1 only B. 2 only C. Both 1 and 2 D. Neither 1 nor 2

Answer - Option A
Explanation: DAWN Mission
Statement 1 is incorrect: DAWN Mission was launched by NASA in 2007.
Statement 2 is correct: It was sent to study two of the three known proto planets of the asteroid belt, Vesta and Ceres. It was the first and only mission to visit both Vesta and Ceres. It was also NASA’s first deep space mission to use ion propulsion.
Dawn mission gave deep insight on understanding about protoplanets of Asteroid belt. It revealed the irregular relief of Vesta in dramatic detail, mapping it from pole to pole while probing it from core to surface.
It also gave key insights into the cryovolcanic activity erupting on the surface of Ceres and a look at an active dwarf planet. Between Ceres and Vesta, Dawn explored about 45% (nearly half) of the mass of the main asteroid belt.
DAWN mission was extended 11 years more than its expected life span. Other Facts
PROTO-PLANET: Proto-planets are small celestial objects that are the size of a moon or a bit bigger. They are small planets, like an even smaller version of a dwarf planet. Astronomers believe that these objects form during the creation of a solar system.
7. Which among the following are human space missions? 1. Orion Exploration 2. Gaganyaan 3. Chandrayaan-2 4. Shenzhou Select the correct answer using the code given below:
    A. 1 and 2 only B. 2 and 3 only C. 1, 2 and 4 only D. 1, 2, 3 and 4

Answer - Option C
Explanation:
Statement 1 is correct: The Orion Multipurpose Crew Vehicle (MPCV) or Orion Exploration-1 is a NASA spacecraft designed to take a crew of up to six Astronauts to destinations beyond Low Earth Orbit including the Moon, Mars and Asteroids.
Orion originated in the Vision for Space Exploration announced in 2004 by President George W. Bush. Initially, the craft was known as CEV – Crew Exploration Vehicle. The project is still under progress.
Statement 2 is correct: Gaganyaan (“Skycraft”) is India’s autonomous 3.7-tonne spacecraft designed to carry a 3-member Indian crew to orbit and safely return to the Earth after a mission duration of few orbits and up to seven days.
Key facts about Gaganyaan
The Indian astronaut would be stationed at an altitude of 400 km from earth for 5-7 days.
They will orbit in a plane perpendicular to equatorial plane.
The payload consists of a crew module and a service module.
It will be lauched using GSLV (cryogenic technology) vehicles
The space vehicle is planned to be launched on ISRO’s GSLV Mk III in 2022.
The flight would take 16 minutes to reach its orbit and during the period of stay of experiments, particularly microgravity experiments.
If successful, it would make India only the fourth member (after Russia, the US, and China) of an elite club of nations with indigenous manned space programmes.
Chandrayaan-2
Statement 3 is incorrect: This is a India’s planned second mission to the Moon which would be totally indigenous mission comprising of an Orbiter, Lander and Rover.
The payloads will collect scientific information on lunar topography, mineralogy, elemental abundance, lunar exosphere and signatures of hydroxyl and water-ice
Shenzhouprogram :
Statement 4 is correct: The Shenzhou program was a manned spaceflight program by the People’s Republic of China. The program put the first Chinese citizen into orbit on 15 October 2003.
8. Recently, a Country has unveiled replica of its future permanently crewed space station “Heavenly Palace” which will replace International Space Station. The country is:
    A. U.S.A B. India C. Japan D. China

Answer - Option D
Explanation:
China has unveiled a replica of its first permanently-crewed space station, which would replace the international community’s orbiting laboratory- the International Space Station (ISS) and symbolises the country’s major ambitions beyond Earth.
About China’s space station:
It is a 17-metre core module. Three astronauts will be permanently stationed in the 60-tonne orbiting lab, which will enable the crew to conduct biological and microgravity research.
Assembly is expected to be completed around 2022 and the station would have a lifespan of around 10 years.
Significance:
The International Space Station – a collaboration between the United States, Russia, Canada, Europe and Japan – has been in operation since 1998 and is due to be retired in 2024.
China will then have the only space station in orbit, though it will be much smaller than the ISS which weighs 400 tonnes and is as large as a football pitch.
9. Consider the following statements about GSAT-29 1. It is a multi-beam and multi-band communication satellite. 2. It has been launched recently onboard PSLV C-41. Which of the above statements is/are correct:
    A. 1 Only B. 2 Only C. Both 1 and 2 D. None

Answer - Option A
10. With reference to the Quadricycles, which of the following statements is/are correct? 1. Quadricycle is a four tyred vehicle of the size of a 3-wheeler but it is fully covered like a car and has an engine like that of a 3-wheeler. 2. Quadricycles were only allowed for nontransport usage under the Motor Vehicles Act 1988, but now has been made usable for transport also. Select the correct answer using the code given below:
    A. 1 only B. 2 only C. Both 1 and 2 D. Neither 1 nor 2

Answer - Option A
Explanation:
1. Quadricycle is a four tyred vehicle of the size of a 3-wheeler but it is fully covered like a car and has an engine like that of a 3-wheeler.
2. Quadricycles were only allowed for nontransport usage under the Motor Vehicles Act 1988, but now has been made usable for transport also.
Select the correct answer using the code given below:
1. Consider the following statements about GSLV Mk III Launch Vehicle: 1. It is a 3 stage heavy lift launch vehicle developed by ISRO. 2. Its first stage has solid propellant, second stage has liquid propellant and third stage has cryogenic engine. Which of the above statements is/are correct:
    A. 1 Only B. 2 Only C. Both 1 and 2 D. None

Answer - Option C
Explanation:
About GSLV Mk III:
GSLV Mk III is a three-stage heavy lift launch vehicle developed by the Indian Space Research Organisation (ISRO).
Two massive boosters with solid propellant constitute the first stage, the core with liquid propellant form the second stage and the cryogenic engine completes the final stage.
2. Consider the following statements about HYSIS: 1. It is an earth observation satellite launched onboard PSLV-C43. 2. It has Hyperspectral Camera which will enable earth observation in visible, near infrared and shortwave infrared regions of electromagnetic spectrum. Which of the above statements is/are correct:
    A. 1 Only B. 2 Only C. Both 1 and 2 D. None

Answer - Option C
Explanation:
Hyper Spectral Imaging Satellite (HysIS) It is an earth observation satellite built around ISRO’s Mini Satellite 2 (IMS-2) bus weighing about 380 kg.
The life of the satellite is five years.
The Indian Space Research Organization’s (ISRO) Polar Satellite Launch Vehicle (PSLV-C43) successfully launched 31 satellites from Satish Dhawan Space Centre (SDSC) in Sriharikota.
About HysIS and its significance:
The primary goal of HysIS is to study the Earth’s surface in visible, near-infrared and shortwave infrared regions of the electromagnetic spectrum.
HysIS will be ISRO’s first full-scale working satellite with this capability. While the technology has been around, not many space agencies have working satellites with hyperspectral imaging cameras as yet.
A hyperspectral imaging camera in space can provide well-defined images that can help to identify objects on Earth far more clearly than regular optical or remote sensing cameras.
The technology will be an added advantage of watching over India from space for a variety of purposes such as defence, agriculture, land use, minerals and so on.
3. Consider the following statements related to the missiles developed under Integrated Guided Missile Development Programme: 1. Prithvi - Surface to Surface short range cruise missile 2. Dhanush - Naval variant of Prithvi Missile 3. Akash - Short range Surface to Air Missile 4. Shaurya- Short range Surface to Surface ballistic missile. Which of the above statements are correct?
    A. 1 and 2 only B. 2, 3 and 4 only C. 2 and 4 only D. 1, 2, 3 and 4

Answer - Option C
Explanation:
Statement 1 is incorrect:
Prithvi - Surface to surface short range ballistic missile,
Trishul - Short range surface to air missile.
Statement 3 is incorrect: Akash medium range mobile surface to air defence system.
Statement 2 is correct: Dhanush- Naval variant of Prithvi Missile.
Statement 4 is correct: Shaurya- Short range surface to surface ballistic missile.
4. The term ‘Data Mining’ is often associated with which of the following:
    A. Cloud computing B. Big data C. Net neutrality D. Internet of Things

Answer - Option B
Explanation:
Big data analytics:
Option b is correct: Data mining is the process of sorting through large data sets to identify patterns and establish relationships to solve problems through data analysis. Data mining tools allow enterprises to predict future trends.
“Data mining” is in fact a misnomer, because the goal is the extraction of patterns and knowledge from large amounts of data, not the extraction (mining) of data itself.
Data mining techniques are used in many research areas, including mathematics, cybernetics, genetics and marketing.
While data mining techniques are a means to drive efficiencies and predict customer behavior, if used correctly, a business can set itself apart from its competition through the use of predictive analysis.
5. NIPUN is related to which of the following?
    A. Entrepreneurship programme of Government of Rajasthan. B. Skill Development Programme of Government of India. C. Cloud service under Digital India initiative. D. E-learning portal for Delhi Police personnel.

Answer - Option D
Explanation:
Delhi Police has launched an e-learning portal ‘Nipun’ to impart training and provide up to date information to police officers.
NIPUN:
The initiative is aimed at imparting in-service online training and information to police personnel through specialised courses available online which have been designed by experts in various fields.
The online courses on this portal have been designed in collaboration with University Grant Commission (UGC), FICCI, NHRC, NCPCR and Janaki Devi Memorial College, Delhi University under project CLAP “The Collaborative Learning and Partnership”.
It will have online courses and resources on law, standing orders, investigation checklists, forms for case files, latest High Court and Supreme Court rulings. It is envisioned that Investigating Officers (IOS) will be able to take benefit of this information during course of their duty or while handling case
6. SIMBEX is a bilateral maritime exercise between India and:
    A. Seychelles B. Sri Lanka C. Singapore D. South Africa

Answer - Option C
Explanation:
SIMBEX – Singapore India Maritime Bilateral Exercise is a naval exercise between two navies in the Andaman Sea and Bay of Bengal.
7. Consider the following statements about Border Area development Programme (BADP): 1. BADP is implemented by Ministry of Defence. 2. The BADP covers North Eastern states of India only. Which of the above statements is/are correct?
    A. 1 only B. 2 only C. Both 1 and 2 D. Neither 1 nor 2

Answer - Option D
Explanation:
(BADP)
Statement 1 is incorrect: Department of Border Management, Ministry of Home Affairs is the implementing agency of Border Area Development Programme (BADP) through the State Governments as part of a comprehensive approach to Border Management.
The programme aims to meet the special development needs of the people living in remote and inaccessible areas situated near the international border and to saturate the border areas with the essential infrastructure through convergence of Central/State/BADP/Local schemes and participatory approach.
BADP was initiated in the border areas of the western region during that Seventh Five Year Plan period for ensuring balanced development of border areas through development of infrastructure and promotion of a sense of security among the border population.
Statement 2 is incorrect: The programme now covers 394 border blocks of 111 border districts in 17 States.
8. Recently, Central Government has announced that a stadium of international standard for differently-abled persons will be set up in:
    A. Manipur B. Meghalaya C. Mizoram D. Assam

Answer - Option B
9. Dharma Guardian is a joint military exercise between India and:
    A. Sri Lanka B. Cambodia C. Japan D. South Korea

Answer - Option C
10. Samudra Shakti is Naval Exercise between India and
    A. South Korea B. U.S C. Japan D. Indonesia

Answer - Option D
1. Consider the following statements: 1. The depletion of ozone layer above the Antarctic region is called as Ozonehole. 2. Ozone is a molecule comprised of three Chlorine atoms that does notoccur naturally. Which of the above statements is/are correct?
    A. 1 only B. 2 only C. Both 1 and 2 D. Neither 1 nor 2

Answer - Option A
Explanation:
A layer of ozone envelops the Earth and keeps damaging ultraviolet, or UV, radiation from reaching living things on the planet’s surface.
The ozone layer exists mainly in the stratosphere, a layer of the atmosphere that reaches from 10 to 50 kilometers (about 6 to 30 miles) above the Earth’s surface.
Ozone is a molecule comprised of three oxygen atoms that occurs naturally in small amounts.
What is ozone hole?
The ozone hole is a region of depleted layers of ozone above the Antarctic region, whose creation is linked to increased cases of skin cancer.
Factors responsible for the depletion of ozone:
Depletion of ozone is due to many factors, the most dominant of which is the release of chlorine from CFCs (Chlorofluorocarbons) which destroys the ozone. CFCs are released by products such as hairsprays, old refrigerators etc.
2. Consider the following statements regarding global efforts for mitigating Marine pollution: 1. MARPOL treaty is signed under the aegis of United Nation Law of Convention of Sea. 2. Emission Control Areas (ECA) are declared under MARPOL Treaty. 3. There are no ECAs in the Indian Ocean region. Which of the above statements is/are correct?
    A. 1 and 2 only B. 3 only C. 2 only D. 1, 2 and 3

Answer - Option D
Explanation:
The International Convention for the Prevention of Pollution from Ships (MARPOL) is an important international convention covering prevention of pollution of the marine environment by ships from operational or accidental causes.
Statement 1 is correct: The MARPOL Convention was adopted on 2 November 1973 by IMO (International Maritime Organistion) in response to a spate of tanker accidents in 1976-1977 under the aegis of UNCLOS.
In 1997, the Protocol was adopted to amended and a new Annex VI (regarding ECAs) was added which entered into force on 19 May 2005.
MARPOL has been updated by amendments through the years.
Law of the Sea treaty, is an international agreement that resulted from the third United Nations Conference on the Law of the Sea (UNCLOS III), which took place between 1973 and 1982.
The Convention defines the rights and responsibilities of nations with respect to their use of the world’s oceans, establishing guidelines for businesses, the environment, and the management of marine natural resource
The Convention includes regulations aime at preventing and minimizing pollution from ships - both accidental pollution and that from routine operations - and currently includes six technical Annexes.
Statement 2 is correct: Emission Control Areas (ECAs), are sea areas in which stricter controls are established to minimize airborne emissions from ships as defined by Annex VI of the 1997 MARPOL Protocol. The emissions specifically comprise SOx, NOx, ODSs (Ozone Depleting Substance) and VOCs (volatile organic compounds) and the regulations came into effect in May 2005
Statement 3 is correct: As of 2018 there are four ECAs: the Baltic Sea, the North Sea, the North American ECA, including most of USA and
Canadian coast and the US Caribbean ECA(none in Indian ocean region however there has been demands for the inclusion of Bay of Bengal )
IMO
IMO is a specialized agency of the United Nations
It is a global standard-setting authority for the safety, security and environmental performance of international shipping.
Its main role is to create a regulatory framework for the shipping industry that is fair and effective, universally adopted and universally implemented.
3. The IUCN Red List status of Hog Deer is:
    A. Endangered B. Critically Endangered C. Least Concern D. Extinct

Answer - Option A
Explanation:
Indian scientists have discovered in India an endangered sub-species of hog deer (Axis porcinus annamiticus), earlier believed to be confined to the eastern part of central Thailand.
Key facts:
Researchers reported the presence of a small population of hog deer in Keibul Lamjao National Park (KLNP), Manipur. The population genetically resembles A. p. annamiticus. The study indicates that the western limit of hog deer is Manipur; not central Thailand as believed.
Significance: Since hog deer is losing habitat in other countries, the genetically distinct and evolutionarily significant population found in KLNP— considered a biodiversity hotspot on the India-Myanmar border— is significant for conservation.
The hog deer or Pada is an endangered species in the IUCN Red List and is protected under Schedule I of the Indian Wild Life (Protection) Act, 1972. The species has lost ground in most of its distribution range. A small and isolated population of under 250 was reported from Cambodia. However, it was widely distributed throughout the Southeast Asian countries at the beginning of the 20th century.
Two sub-species of hog deer have been reported from its range. The western race is distributed from Pakistan and the terai grasslands (along the Himalayan foothills, from Punjab to Arunachal Pradesh), while the eastern race of hog deer is found in Thailand, Indo-China, Laos, Cambodia, and Vietnam.
4. INDRA 2018 is a join military exercise between India and:
    A. Rwanda B. Russia C. Japan D. Belgium

Answer - Option B
5. The “Bhungroo” technique recently seen in news, is related to:
    A. Water harvesting B. Indigenous Cow breeding C. Defence equipment D. Genome Editing

Answer - Option A
Explanation:
Bhungroo is a water management system that injects and stores excess rainfall underground and lifts it out for use in dry spells. Adoption of this technology has decreased salt deposits on soil and increased fresh water supply, saving farmers from drought.
6. Recently the extinct species Vorombe Titan has been given the title of ‘World’s Largest Bird’. The species was indigenous to:
    A. Antarctic Region B. Sahara Region C. Mediterranean Region D. Madagascar Region

Answer - Option D
Explanation:
Elephant birds (belonging to the family Aepyornithidae) are an extinct group of colossal flightless birds that roamed Madagascar during the Late Quaternary.
Vorombe Titan was about three metres tall and weighed up to 800 kg
7. ‘Annualized Slippage Ratio’ is the:
    A. Ratio of bad loans to total loans. B. Percentage of fresh NPAs as percentage of standard advances C. Amount a bank expects to lose due to credit risks D. Percentage of money a bank has set aside to cover for bad loans.

Answer - Option B
Explanation:
Ratio is the percentage of fresh NPAs as percentage of standard advances at the beginning of the quarter. In other words, slippage ratio is calculated as the addition of Gross NPA’s during the year as a percentage of outstanding standard assets of the previous year. It is an important indicator of asset quality.
The gross nonperforming asset ratio, slippage ratio, credit cost, provision coverage ratio all part of a long list of indicators that banking analysts are tracking to understand how sound Indian banks are. Recently another indicator the divergence ratio has been introduced to understand the banking health of India.
8. The “Breteau Index”, recently seen in news, is related to:
    A. Measure of soil biodiversity B. Measure of marine debris C. Immigration estimatin D. Vector surveillance

Answer - Option D
Explanation:
Breteau Index
Vector surveillance: Surveillance on Aedes aegypti density is important in determining factors related to dengue transmission, in order to prioritize areas and seasons for vector control. Selection of appropriate surveillance strategies are based upon outcome/objective, also taking into consideration time, resources, and infestation levels.
9. “BASIC”, a group of nations, is related to:
    A. Research expedition in Arctic B. WTO reforms C. Permanent Membership to UN Security Council D. Climate Change Negotiations

Answer - Option D
Explanation:
It is a bloc of four developing countries (Brazil, South Africa, India and China), formed by an agreement to act jointly at the COP 15-Copenhagen Climate Summit (CCS) in the year 2009. The alliance was formed to negotiate with developed countries regarding the further course of action to be taken on climate change after the end of first mandate of Kyoto Protocol.
Subsequently, the CCS resulted in Copenhagen Accord between US and BASIC countries. Since then, the BASIC countries have met regularly and address the common concerns and interests of the developing countries vis-a-vis developed countries regarding climate change.
10. Which committee has recommended the Public Credit Registry?
    A. Raja Chelliah committee B. Y.M. Deosthalee committee C. A C Shah committee D. Abid Hussain committee

Answer - Option B

Other Articles



shape Job-Alerts

Competitive Exams - Recent Job Notifications
Category
Banking SSC Railway
Defence Police Insurance
Click Here For – All India Latest Jobs

shape SP Quiz

Competitive Exams - Practice Sets
Category Quiz
English Language Spotting Errors
Current Affairs Current Affairs
Quantitative Aptitude Data Interpretation